Difference between revisions of "2009 AIME I Problems/Problem 5"

(Solution)
m (Solution)
Line 4: Line 4:
  
 
== Solution ==
 
== Solution ==
{{image}}
+
<center><asy>
 +
import markers;
 +
defaultpen(fontsize(8));
 +
size(300);
 +
pair A=(0,0), B=(30*sqrt(331),0), C, K, L, M, P;
 +
C = intersectionpoints(Circle(A,450), Circle(B,300))[0];
 +
K =  midpoint(A--C);
 +
L = (3*B+2*A)/5;
 +
P = extension(B,K,C,L);
 +
M = 2*K-P;
 +
draw(A--B--C--cycle);
 +
draw(C--L);draw(B--M--A);
 +
markangle(n=1,radius=15,A,C,L,marker(markinterval(stickframe(n=1),true)));
 +
markangle(n=1,radius=15,L,C,B,marker(markinterval(stickframe(n=1),true)));
 +
dot(A^^B^^C^^K^^L^^M^^P);
 +
label("$A$",A,(-1,-1));label("$B$",B,(1,-1));label("$C$",C,(1,1));
 +
label("$K$",K,(0,2));label("$L$",L,(0,-2));label("$M$",M,(-1,1));
 +
label("$P$",P,(1,1));
 +
label("$180$",(A+M)/2,(-1,0));label("$180$",(P+C)/2,(-1,0));label("$225$",(A+K)/2,(0,2));label("$225$",(K+C)/2,(0,2));
 +
label("$72$",(L+P)/2,(-1,0));label("$300$",(B+C)/2,(1,1));
 +
</asy></center>
  
Since <math>K</math> is the midpoint of <math>\overline{PM}, \overline{AC}</math>.
+
Since <math>K</math> is the midpoint of <math>\overline{PM}</math> and <math>\overline{AC}</math>, quadrilateral <math>AMCP</math> is a parallelogram, which implies <math>AM||LP</math> and  <math>\bigtriangleup{AMB}</math> is similar to <math>\bigtriangleup{LPB}</math>
 
 
Thus, <math>AK=CK,PK=MK</math> and the opposite angles are congruent.
 
 
 
Therefore, <math>\bigtriangleup{AMK}</math> is congruent to <math>\bigtriangleup{CPK}</math> because of SAS
 
 
 
<math>\angle{KMA}</math> is congruent to <math>\angle{KPC}</math> because of CPCTC
 
 
 
That shows <math>\overline{AM}</math> is parallel to <math>\overline{CP}</math> (also <math>CL</math>)
 
 
 
That makes <math>\bigtriangleup{AMB}</math> similar to <math>\bigtriangleup{LPB}</math>
 
  
 
Thus,
 
Thus,

Revision as of 17:22, 7 August 2010

Problem

Triangle $ABC$ has $AC = 450$ and $BC = 300$. Points $K$ and $L$ are located on $\overline{AC}$ and $\overline{AB}$ respectively so that $AK = CK$, and $\overline{CL}$ is the angle bisector of angle $C$. Let $P$ be the point of intersection of $\overline{BK}$ and $\overline{CL}$, and let $M$ be the point on line $BK$ for which $K$ is the midpoint of $\overline{PM}$. If $AM = 180$, find $LP$.

Solution

[asy] import markers; defaultpen(fontsize(8)); size(300); pair A=(0,0), B=(30*sqrt(331),0), C, K, L, M, P; C = intersectionpoints(Circle(A,450), Circle(B,300))[0]; K =  midpoint(A--C); L = (3*B+2*A)/5; P = extension(B,K,C,L); M = 2*K-P; draw(A--B--C--cycle); draw(C--L);draw(B--M--A); markangle(n=1,radius=15,A,C,L,marker(markinterval(stickframe(n=1),true))); markangle(n=1,radius=15,L,C,B,marker(markinterval(stickframe(n=1),true))); dot(A^^B^^C^^K^^L^^M^^P); label("$A$",A,(-1,-1));label("$B$",B,(1,-1));label("$C$",C,(1,1)); label("$K$",K,(0,2));label("$L$",L,(0,-2));label("$M$",M,(-1,1)); label("$P$",P,(1,1)); label("$180$",(A+M)/2,(-1,0));label("$180$",(P+C)/2,(-1,0));label("$225$",(A+K)/2,(0,2));label("$225$",(K+C)/2,(0,2)); label("$72$",(L+P)/2,(-1,0));label("$300$",(B+C)/2,(1,1)); [/asy]

Since $K$ is the midpoint of $\overline{PM}$ and $\overline{AC}$, quadrilateral $AMCP$ is a parallelogram, which implies $AM||LP$ and $\bigtriangleup{AMB}$ is similar to $\bigtriangleup{LPB}$

Thus,

\[\frac {AM}{LP}=\frac {AB}{LB}=\frac {AL+LB}{LB}=\frac {AL}{LB}+1\]

Now lets apply the angle bisector theorem.

\[\frac {AL}{LB}=\frac {AC}{BC}=\frac {450}{300}=\frac {3}{2}\]

\[\frac {AM}{LP}=\frac {AL}{LB}+1=\frac {5}{2}\]

\[\frac {180}{LP}=\frac {5}{2}\]

\[LP=\boxed {072}\]

See also

2009 AIME I (ProblemsAnswer KeyResources)
Preceded by
Problem 4
Followed by
Problem 6
1 2 3 4 5 6 7 8 9 10 11 12 13 14 15
All AIME Problems and Solutions